You are on page 1of 30

TARGET EDUCARE PVT. LTD.

AIPGEE - 1 Explanations
MASTER KEY (Errata – Marked red in color)
Q A Reference Q A Reference Q A Reference Q A Reference
1 4 SAT-1 51 1 SAT-8 101 1 SAT-15 151 1 SAT-22
2 4 SAT-1 52 3 SAT-8 102 2 SAT-15 152 2 SAT-22
3 2 SAT-1 53 2 SAT-8 103 3 SAT-15 153 1 SAT-22
4 3 SAT-1 54 4 SAT-8 104 3 SAT-15 154 1 SAT-22
5 1 SAT-1 55 2 SAT-8 105 3 SAT-15 155 1 SAT-23
6 4 SAT-1 56 1 SAT-8 106 1 SAT-16 156 4 SAT-23
7 4 SAT-1 57 3 SAT-9 107 1 SAT-16 157 2 SAT-23
8 1 SAT-2 58 1 SAT-9 108 1 SAT-16 158 3 SAT-23
9 2 SAT-2 59 4 SAT-9 109 2 SAT-16 159 4 SAT-23
10 3 SAT-2 60 2 SAT-9 110 2 SAT-16 160 4 SAT-23
11 1 SAT-2 61 4 SAT-9 111 3 SAT-16 161 4 SAT-23
12 4 SAT-2 62 2 SAT-9 112 1 SAT-16 162 3 SAT-24
13 1 SAT-2 63 2 SAT-9 113 4 SAT-17 163 1 SAT-24
14 1 SAT-2 64 3 SAT-10 114 3 SAT-17 164 1 SAT-24
15 3 SAT-3 65 2 SAT-10 115 1 SAT-17 165 2 SAT-24
16 2 SAT-3 66 2 SAT-10 116 3 SAT-17 166 2 SAT-24
17 2 SAT-3 67 4 SAT-10 117 2 SAT-17 167 2 SAT-24
18 3 SAT-3 68 3 SAT-10 118 3 SAT-17 168 3 SAT-24
19 1 SAT-3 69 1 SAT-10 119 4 SAT-17 169 4 SAT-25
20 4 SAT-3 70 3 SAT-10 120 4 SAT-18 170 4 SAT-25
21 3 SAT-3 71 1 SAT-11 121 4 SAT-18 171 1 SAT-25
22 4 SAT-4 72 3 SAT-11 122 2 SAT-18 172 3 SAT-25
23 2 SAT-4 73 4 (1 & 3) SAT-11 123 1 SAT-18 173 3 SAT-25
24 4 SAT-4 74 4 SAT-11 124 3 SAT-18 174 4 SAT-25
25 1 SAT-4 75 1 SAT-11 125 4 SAT-18 175 3 SAT-25
26 1 SAT-4 76 1 SAT-11 126 1 SAT-18 176 3 SAT-26
27 3 SAT-4 77 4 SAT-11 127 4 SAT-19 177 2 SAT-26
28 2 SAT-4 78 2 SAT-12 128 2 SAT-19 178 3 SAT-26
29 1 SAT-5 79 2 SAT-12 129 4 SAT-19 179 4 SAT-26
30 2 SAT-5 80 4 SAT-12 130 3 SAT-19 180 3 SAT-26
31 1 SAT-5 81 1 SAT-12 131 4 SAT-19 181 4 SAT-26
32 2 SAT-5 82 1 SAT-12 132 3 SAT-19 182 2 SAT-26
33 2 SAT-5 83 1 SAT-12 133 4 SAT-19 183 1 SAT-27
34 3 SAT-5 84 4 SAT-12 134 3 SAT-20 184 3 SAT-27
35 2 SAT-5 85 1 SAT-13 135 3 SAT-20 185 3 SAT-27
36 4 SAT-6 86 3 SAT-13 136 4 SAT-20 186 4 SAT-27
37 4 SAT-6 87 2 SAT-13 137 3 SAT-20 187 2 SAT-27
38 4 SAT-6 88 3 SAT-13 138 4 SAT-20 188 3 SAT-27
39 2 SAT-6 89 3 SAT-13 139 3 SAT-20 189 3 SAT-27
40 2 SAT-6 90 1 SAT-13 140 4 SAT-20 190 1 SAT-28
41 3 SAT-6 91 1 SAT-13 141 2 SAT-21 191 2 SAT-28
42 2 SAT-6 92 4 SAT-14 142 2 SAT-21 192 1 SAT-28
43 2 SAT-7 93 1 SAT-14 143 4 SAT-21 193 4 SAT-28
44 3 SAT-7 94 3 SAT-14 144 3 SAT-21 194 4 SAT-28
45 3 SAT-7 95 1 SAT-14 145 4 SAT-21 195 2 SAT-28
46 1 SAT-7 96 4 SAT-14 146 3 SAT-21 196 2 SAT-29
47 1 SAT-7 97 3 SAT-14 147 2 SAT-21 197 4 SAT-29
48 2 SAT-7 98 2 SAT-14 148 1 SAT-22 198 2 SAT-29
49 2 SAT-7 99 2 SAT-15 149 4 SAT-22 199 2 SAT-29
50 1 SAT-8 100 3 SAT-15 150 2 SAT-22 200 3 SAT-29

www.targeteducare.com info@targeteducare.com
Helpline: 93 2233 6677 (Working days 11 AM to 6 PM)
TARGET EDUCARE PVT. LTD.
AIPGEE - 1 Explanations
Ans.1
Piloerection is not important in human beings. It means hairs ‘standing on end’. Sympathetic stimulation causes the arrector pilli
muscles attached to the hair follicles to contract which brings the hairs to an upright stance. This is not important in human beings but
in lower animals, upright protections of hairs allows them to entrap a thick layers of ‘insulator air’ next to the skin, so that transfer of
heat to the surroundings is greatly depressed’.
Temperature increasing mechanisms (when the body is too Temperature decreasing mechanisma (when the body is
cold) too hot)
- vasoconstriction of skin blood vessels (stimulation of - Vasodilatation of skin blood vessels.(inhibition of
posterior hypothalamic sympathetic centers) posterior hypothalamic sympathetic centers)
- Increase in thermogenesis (heat production). - Decrease in thermogenesis (heat production)-inhibition
- Enhanced shivering, thyroxine secretion, Sympathetic of shivering, inhibition of chemicakl thermogenesis.
excitation of heat production - Sweating: (heat loss)- evaporative heat loss.
- Piloerection: (conserve heat) not important in human
beings, important in lower animals.

Ans.2
Massage and the application of liniments to painful area relieves pain due to inhibition of pain pathway in the dorsal horn gate by
stimulation of large diameter touch pressure afferent fibers.
Touch and pressure is carried by Aβ sensory fibers and pain is carried by Aδand C type fibers. The diameter of Aβ > Aδ > C.
Many people have learned from practical experience that touching or shaking an injured area decrease the pain of injury. The
laminiscal fibers carrying tactile senses give collaterals to pain carrying afferent nerve fibers. When leminiscal fibers are stimulated by
tactile sensation they cause presynaptic inihibitionQ on the primary pain carrying afferent and pain is inhibited.

Ans.3
Hypothalamic regulation of the apetite for food depends primarily on the interaction of two areas: a lateral ‘feeding center’ in the bed
nucleus of the medial forebrain bundle at its junction with the pallidohypothalamic fibers, and a medial ‘satiety center’ in the
ventromedial nucleus. Stimulation of the feeding center evokes eating behavior in conscious animals, and its destruction causes
severe, fatal anorexia in otherwise healthy animals. Stimulation of the ventromedial nucleus causes cessation of eating, whereas
lesions in this region cause hyperphagia and, if the food supply I abundant, the syndrome of hypothalamic obesity.

Ans.4
Compliance of lung is a measure of stretchability of lungs and hence its total capacity. It is the change in lung volume per unit change
in air way pressure (V/P).
Lung Compliance is increased in – emphysema
Lung compliance decreased in-
- Deformities of thorax eg- kyphosis, scoliosis.
- Paralysis of respiratory muscles.
- Pleural effusion.
- Abnormal thorax eg- pneumothorax, hydrothorax, hemothorax.
- Interstitial pulmonary fibrosis.
- Interstitial lung disease.
- Pulmonary congestion.

Ans.5
Deoxygenated hemoglobin binds more H+ than oxyhemoglobin does and forms carbamino compounds more readily, binding of
oxygen to hemoglobin reduces its affinity for carbon dioxide (Haldane effect). Consequently, venous blood carries more carbon
dioxide than arterial blood, carbon dioxide uptake is facilitated in the tissues, and CO2 release is facilitated in the lungs. About 11%of
the CO2 added to the blood in the systemic capillaries is carried to the lungs as carbamino CO2.

Ans.6
The overall blood flow in circulation of adult at rest is about 5 liter/ minute which equals to cardiac output because it is the amount of
blood pumped by heart per unit time. So
Blood pressure = Cardiac output X peripheral resistance or blood flow (Q) X resistance ®
Venous return = MSFP-RAP/RVR
Where, MSFP = Mean systemic filling pressure
RAP = Right atrial pressure
RVR = Resistance to venous return.

www.targeteducare.com info@targeteducare.com
Helpline: 93 2233 6677 (Working days 11 AM to 6 PM)
TARGET EDUCARE PVT. LTD.
AIPGEE - 1 Explanations
Ans.8
Sources of digestive enzymes-
- Salivary gland- alpha amylase Exocrine pancreas- Trypsin, Chymotrypsin, elastase, pancreatic lipase,
- Lingual glands- lipase bile salt acid lipase, colipase, cholestery ester hydrolase
- Stomach- Pepsinogen and lipase Pancreatic alpha amylase, carboxy peptidase A and B, Phospholipase A2,
- Cytoplasm of mucosal cells- peptidase Ribonuclease, Deoxyribonuclease.
- Intestinal mucosa- Enteropeptidase,
Aminopeptidase, Carboxypeptidase,
Endopeptidase, Dipeptidase, Maltase,
Lactase, Sucrase, alpha dextrinase,
trehalase, nuclease.

Ans.9
Pancreatic juice is essentially alkaline with a pH as high as 8.0 to 8.3
Intestinal juice pH
Saliva 6.0 to 7.0
Gastric juice 1.0 to 3.5
Pancretic juice 8.0 to 8.3
Bile 7.8

Ans.11
Negatively charged large molecules are filtered less easily than positively charged molecules of equal molecular size, and even neutral
molecule are filtered much more easily than negatively charged molecules of same size. This is because, the negative charges of
basement membrane and podocytes restrict and repel large negatively charged and attract positively charged molecules. This explains,
only 0.2% filterability of negatively charged albumin with an effective diameter of ~ 7 nm.
Inulin (a polymer of fructose) that is freely filtered, neither reabsorbed nor secreted in tubules, and creatinine (that is freely filtered,
secreted and reabsorbed in some amount) are used to measure glomerular filtration rate (GFR).

Ans.12
The sertoli cells secrete androgen bonding protein (ABP), inhibin, and MIS i.e. Mullerian inhibiting substance. Sertoli cells have
receptors for FSH and testosterone. After combining with FSH, sertoli cells stimulate the first half of spermatogenesis. Subsequently
testosterone- sertoli cells binding causes development of last half of spermatogenesis.

Ans.13
Oxytocin causes contraction of the myoepithelial cells lining the duct walls, with consequent ejection of the milk through the nipple.
The reflex release of oxytocin initiated by touching the nipples and areolas (milk ejection reflex).
Suckling not only evokes reflex oxytocin release and milk ejection; it also maintains and augments the secretion of milk because of
the stimulation of prolactin secretion produced by suckling.
Oxytocin also cause contraction of smooth muscles of the uterus.

Ans.14
The sequence of cell cycle is-
G0 phase- Quiscent cells are in G0 phase or resting phase.
G1 phase (12 hrs)-presynthetic phase.
S phase(6 to 8 hrs)- DNA synthesis phase.
G2 phase (4 to 5 hrs)- (premitotic phase) cytoplasmic enlargement; preparation for cell division; DNA repair can occur.
M phase- Mitotic phase.

Ans.15
Chemical mediators of Inflammation
1. Preformed mediator in secretory granules : Histamine
: Serotonin
: Lysosomal enzyme
2. Newly synthesized : Prostaglandin leukotrienes, platelet activating factors
: Activated oxygen species
: Nitric oxide
: Cytokines
* Tumor necrosis factor – α
* Interleukin – 1
www.targeteducare.com info@targeteducare.com
Helpline: 93 2233 6677 (Working days 11 AM to 6 PM)
TARGET EDUCARE PVT. LTD.
AIPGEE - 1 Explanations
3. Factor XII activation : - Kinin system (bradykinin)
: - Coagulation / fibrinolysis system
4. Complement activation : - C3a
Anaphylatoxin
C5a
C3b
C5b-9 (membrane attack complex)

Ans.16
Leukotrienes (LT) or slow-reacting substances of anaphylaxis (SRS-As) are so named as they were first isolated from leucocytes.
Firstly, unstable leukotriene A4 (LTA4) is formed which is acted upon by enzymes to form LTB4 (chemotactic for phagocytic cells and
stimulates phagocytic cell adherence) while LTC4, LTD4 and LTE4 have common actions by causing smooth muscle contraction and
thereby induce vasoconstriction, bronchoconstriction and increased vascular permeability.
C5a is chemotatic for leucocytes.

Ans.17
Mucus cyst of the mouth is a retention type of cyst resulting from blockage or rupture of salivary gland duct. It is most often found on
the lower lip as a result of trauma.

Ans.18
In chronic venous congestion liver, the centre of liver lobule is tan brown with surrounding zones of uncongested liver. This
appearance is called nutmeg liver. Microscopically, there is centrilobular coagulative necrosis with haemorrhage accompanied by
periportal fatty change.

Ans.19
Necrosis is defined as focal death along with degradation of tissue by hydrolytic enzymes liberated by cells. It is invariably
accompanied by inflammatory reaction
Necrosis can be caused by various agents such as hypoxia, chemical and physical agents, microbial agents and immunological injury.
Two essential changes bring about irreversible cell injuries in necrosis- cell digestion by lytic enzymes and denaturation of proteins.
These processes are morphologically identified by characteristic cytoplasmic and nuclear changes in necrotic cell. The cytoplasm
appears homogeneous and intensely eosinophillic. Occasionally, it may show vacuolation or dystrophic calcification. The nuclear
changes include condensation of nuclear chromatin (pyknosis) which may either undergo dissolution (karyolysis) or fragmentation
into many granular clumps (karyorrhexis).

Ans.20
Leprosy has two different patterns of disease. One is characterized by tuberculosis leprosy with effective cell mediated immunity,
asymmetric peripheral nerve involvement and presence of well defined granulomas at lesional site. Other pattern is lepromatous
leprosy characterized by defective immunity, symmetric skin thickening and nodules and presence of large collections of lipid laden
macrophages (Virchows lepra cells) filled with aggregates of acid fast bacilli (globi) at lesional site. These lesions are infections and
may lead to leonine facies. Granulomas are absent from lesional site.

Ans.21
A particular form of gas embolism, called ‘decompression sickness’, occurs when individual are exposed to sudden changes in
atmospheric pressure. A more chronic form of decompression sickness is called ‘Cassion disease’, where persistence of gas emboli in
the bones leads to multiple foci of ischemic necrosis; the heads of the femurs, tibias, and humeri are most commonly affected.

Ans.22
The diagnosis of Hodgkin’s disease rests on identification of RS cells, though uncommonly similar cells can occur in infectious
mononucleosis and other forms of lymphomas. Therefore, additional cellular and architectural features of the biopsy must be given
due consideration for making the histologic diagnosis.
1. Classic RS cell is a large cell which has characteristically a bilobed nucleus appearing as mirror image to each other but
occasionally the nucleus may be multilobed.
2. Lacunar type RS cell is smaller and in addition to above features has a pericellular space or lacuna in which it lies, which is
due to artefactuals shrinkage of the cell cytoplasm. It is characteristically found in nodular sclerosis variety of HD.

Ans.23
Laboratory diagnosis of hypochromic anemias
Test Iron deficiency Chronic disorders Thalassemia Sideroblastic anemia
1.MCV, MCH, MCHC Reduced Low normal to reduced Very low Very low (except MCV raised in

www.targeteducare.com info@targeteducare.com
Helpline: 93 2233 6677 (Working days 11 AM to 6 PM)
TARGET EDUCARE PVT. LTD.
AIPGEE - 1 Explanations
acquired type)
2. Serum iron Reduced Reduced Normal Raised
3. TIBC Raised Reduced Normal Normal
4. Serum ferritin Reduced Raised Normal Raised (complete saturation)
5. Marrow iron stores Absent Present Present Present
6.Iron in normoblasts Absent Absent Present Ring sideroblast
7.Hb elctrophoresis Normal Normal Abnormal Normal
In megaloblastic anemia-
There is rise in serum unconjugated bilirubin and LDH as a result of ineffective erythropoiesis causing marrow cell break down.
The serum iron and ferritin may be normal or elevated.

Ans.24
Prothrombin time depends on factor II, VII, IX and X. the synthesis of these factors depends on vitamin K. Vitamin K deficiency
occurs in liver disorder as well as in obstructive jaundice or fat malabsorption of any kind.

Ans.25
Amoebiasis is caused by Entamoeba histolytica, named for its lytic action on tissues. It is the most important intestinal infection of
man.
The parasite occurs in 2 forms: a trophozoite form which is active adult form seen in the tissues and diarrhoeal stools, and a cystic
form seen in formed stools but not in the tissues.
The lesions of amoebiasis include amoebic liver abscess and spread to other sites.
Amoebic colitis, the most common type of amoebic infection begins as a small area of necrosis of mucosa which may ulcerate. These
ulcerative lesions may enlarge, develop undermining of margins of the ulcer due to lytic action of the trophozoite and have necrotic
bed. Such chronic amoebic ulcers are described as flask shaped or bottle mouth shaped ulcers due to their shape. The margin of the
ulcer shows inflammatory response consisting of admixture of polymorphonuclear as well as mononuclear cells.

Ans.26
SLE is a multisystem autoimmune disease of protean manifestations. Immunologically the disease is associated within enormous array
of autoantibodies, classically including anti nuclear antibodies (ANAs). ANAs are directed against several nuclear antigens-
- Antibodies to dsDNA (The most specific test).
- Antibodies to histones.
- Antibodies to non histone proteins bound to RNA.
- Antibodies to nuclear antigens (The most sensitive test).

Ans.27
Gaucher’s disease results from mutation in the gene that encodes glucosylceramidase. There are five Autosomal recessive variants of
Gaucher disease resulting from distinct allelic mutations. Common to all is variably deficient activity of a glucosylceramidase that
normally cleaves the glucose residue from ceramide. This leads to an accumulation of glucosylceramide in the mononuclear
phagocytic cells and their transformation into so called Gaucher cells.

Ans.28
Hairy cell leukemia (HCL), is an unusual and uncommon form of chronic leukemia in which there is presence of abnormal
mononuclear cells with hairy cytoplasmia projections in the bone marrow, peripheral blood and spleen.
These leukemic hairy cells have characteristically positive cytochemical staining for tartarate resistant acid phosphatase. The
controversy on the origin of the hairy cells whether these cells represents neoplastic T cells, B cells or monocytes, is settled with the
mononuclear analysis of these cells which assigns them B cell origin expressing CD19, CD20 and CD22 antigen. In addition to B cell
markers, hairy cells are also positive for CD11, CD25 and CD103.

Ans.29
Characters Gram positive Gram negative
Thickness Thicker Thinner
Variety of amino acids Few Several
Aromatic and sulphur containing amino acids Absent Present
Lipids Absent or scant Present
Teichoic acid Present Absent

www.targeteducare.com info@targeteducare.com
Helpline: 93 2233 6677 (Working days 11 AM to 6 PM)
TARGET EDUCARE PVT. LTD.
AIPGEE - 1 Explanations
Ans.30
The milk is heated at either 63 degree C for 30 minutes (the holder method) or 72 degree C for 15 to 20 seconds (the flash process)
followed by cooling quickly to 13 degree C or lower. By these processes all non sporing pathogens such as mycobacteria, brucellae
and salmonellae are destroyed. Coxiella burnetti is relatively heat resistant and may survive the holder method.

Ans.31
Chemical disinfectants which can be safely applied to skin or mucus membrane and are used to prevent infection by inhibiting the
growth of bacteria are called antiseptics.

Ans.32
Mycoplasmas are a group of bacteria that are devoid of cell walls and so are highly pleomorphic, with no fixed shape or size. They
lack even cell wall precursors like muramic acid or diaminopimelic acid. The cells are bounded by a soft trilaminar unit membrane
containing sterols. Because of their plasticity, they can pass through bacterial filters and have often been mistaken for viruses.

Ans.33
Phagocytic cells are the mononuclear macrophages (of blood and tissue) and the polymorphonuclear microphages.
The primary function of macrophages is phagocytosis.

Ans.34
Haptens are substances which are incapable of inducing antibody formation by themselves but can react specifically with antibodies.
Haptens may be complex or simple; while complex haptens can precipitate with specific antibodies, simple haptens are non
precipitating. They can inhibit precipitation of specific antibodies by the corresponding antigens or complex hapten.

Ans.35
The major clinical features of the GVH reaction in animals are retardation of growth, emaciation, diarrhoea, hepatosplenomegaly,
lymphoid atrophy and anemia, terminating fatally. The syndrome has been called runt disease.

Ans.36
Pfieffer (1894) discovered that cholera vibrios were lysed when injected intraperitoneally into specifically immunized guinea pigs
(bacteriolysis in vivo or pfieffer’s phenomenon). Bordet (1895) extended these observations and established that immune bacteriolysis
and hemolysis required two factors- the heat stable antibody and a heat labile factor, which was called alexine. This term has been
replaced by the present name complement which was coined by Ehrlich, because this factor complemented the action of antibody.

Ans.37
The antibody formed in the primary response is predominantly IgM and in the secondary response IgG. The early antibody is more
specific but less avid then the late antibody.

Ans.38
Transcobalamin II deficiency is inherited as autosomal recessive patients show metabolic effects of vitamin B12 deficiency including
megaloblastic anemia and intestinal villous atrophy. The associated immunological defects are depleted plasma cells, diminished
immunoglobulin levels and impaired phagocytosis. Treatment with vitamin B12 has been reported to restore hematopoietic,
gastrointestinal and B cell functions but not phagocytic activity.

Ans.39
In HIV the genome is diploid, composed of two identical single stranded, positive sense RNA copies. In association with viral RNA is
the reverse transcriptase enzyme, which is a characteristic feature of retroviruses.

Ans.40
AIDS in the developing countries differs from the disease in the western countries clinically too. In Africa, the major manifestation is
pronounced wasting so that it has been called the ‘slim disease’

Ans.41
Congenital or vertical transmission is quite common from carrier mothers. The risk to babies is high if the mother is HBsAg positive
(10 to 90%) and low if negative (5 to 15%).

Ans.42
HBV is a 42 nm DNA virus with an outer envelop and an inner core, 27 nm in diameter, enclosing the viral genome and a DNA
polymerase.

www.targeteducare.com info@targeteducare.com
Helpline: 93 2233 6677 (Working days 11 AM to 6 PM)
TARGET EDUCARE PVT. LTD.
AIPGEE - 1 Explanations
Ans.43
Properties of choline esters.
Choline ester Hydrolysis by Ach E BuChE Actions Musc. Nico Selective action on
Acetylcholine ++ + + + Non selective
Methacholine + - + ± Cvs
Carbachol - - + + g.i.t., bladder
Bethanechol - - + - g.i.t., bladder

• Amongst given options carbactol has maximum nicotinic action. It has both nicotinic as well as muscarinic actions.
• All other drugs have muscarinic actions.

Cholinomimetic drugs
With Muscarinic actions With Nicotinic actions
a) Acetylcholine a) Acetylcholine
b) Methacholine b) Carbachol
c) Carbactol c) Arecholine
d) Bethanectol
e) Pilocarpine
f) Muscarine
g) Arecholine

Ans.44
Contraindications - Atropinic drugs are absolutely contraindicated in individuals with a narrow iridocorneal angle may precipitate
acute congestive glaucoma. However, marked rise in intraocular tension is rare in patients with wide prostatic hypertrophy – urinary
retention can occur.

Ans.45
Classification – Nonselective (β1 and β2)
a) Without intrinsic sympathomimetic activity Propranololε , Sotalol, Timololε.
b) With intrinsic sympathomimetic activity Pindolol.
c) With additional a blocking property Labetalol, Carvedilol.
– Cardioselective (β1)
Metoprolol, Atenololε, Acebutolol, Bisoprolol, Esmolol, Betaxolol, Celiprolol.

Ans.46
Heart – Ppnl decreases heart rate, force of contraction (at relatively higher doses) and cardiac output (c.o.).
Heart - The most prominent effect of atropine is to cause tachycardia.
Heart- Adr increases heart rate by increasing the slope of slow diastolic depolarization of cells in the SA node.
- CVS Methylxanthines directly stimulate the heart and increase force of myocardial contractions. They tend to increase heart rate
by direct action, but decrease it by causing vagal stimulation.
- Aminophylline is a methylxanthine.

Ans.47
Interactions
1) Alcohol synergises with tranquilizers, antidepressants, antihistaminics, hypnotics, opioide  marked CNS depression with
motor impairment can occur: Chances ofaccidents increase.
2) Sulfonylureas (specially chlorpropamide), certain cephalosporins (cefoperazone, moxalactam, cefamandole) and
metronidazole: individuals on these drugs have experienced bizarre, someshat disulfiram like reactions when thy consume
alcohol.
3) Acute alcohol ingestion inhibits, while chronic intake induces tolbutamide, phenytoin (and many other drugs) metabolism.
4) Insulin and sulfonylureas; alcohol enhances hypoglycaemia acutely.
5) Aspirin causes more gastric bleeding when taken with alcohol.
6) Alcoholics are more prone to paracetamol toxicity due to enhanced generation of its toxic metabolite.

Ans.48
• Tolerance is exhibited to most actions of morphine except for miosis and constipation.
• Morphine is a centrally acting analgesic. It has site specific depressant and stimulant action in CNS.
• Morphine produces pronounce psychological and physical dependence. Its abuse liability is very high.

www.targeteducare.com info@targeteducare.com
Helpline: 93 2233 6677 (Working days 11 AM to 6 PM)
TARGET EDUCARE PVT. LTD.
AIPGEE - 1 Explanations
Ans.49
I. Bronchodilators
A. Sympathomimetics: Adrenaline, Ephedrine, Isoprenaline, Salbutamolε, Terbutaline, Bambuterol, Salmeterol, Formoterol.
B. Methylxanthines: Theophylline (anhydrous)ε, Aminophyllineε, Choline theophyllinate, Hydroxyethyl theophylline,
Theophylline ethanolate of piperazine.
C. Anticholinergics: Atropine methonitrate, Ipratropium bromide, Tiotropium bromide.

II. Leukotriene antagonists


Montelukast, Zafirlukast.

III. Mast cell stabilizers


Sodium cromoglycate, Nedocromil, Ketotifen.
IV. Corticosteroids
A. Systemic: Hydrocortisone, Prednisolone and others.
B. Inhalational: Beclomethasone dipropionateε, Budesonide, Fluticasone propionate, Flunisolide.

• Organophosphates have cholinergic action, can cause exacerbation of asthma.

Ans.50
• Pharmacokinetics is the quantitative study of drug movement in, through and out of the body.
• Study of drug effect is Pharmacodynamics
Dose of drug
administered

Drug concentration in DISTRIBUTION Pharmacokinetics


systemic circulation Drug in tissues of distribution

ELIMINATION
Drug concentration at
site of action Drug metabolized or excreted

Pharmacologic effect Pharmacodynamics

Clinical response

>Toxicity Efficacy
Remember
• Pharmacodynamics (D) → Drug does to body
• Pharmacokinetics → Body does to drug.

Ans.51
• Specificity refers to the ability of a drug to bind specific receptor and produce a selective action so that side effects do not
occur.
• High specificity decreases the side effects.
• “the pharmacologist who produces a new drug and the doctor who gives, it to a patient share the desire that it should possess
a selective action, so that additional and unwanted adverse effects do not complication the management of the patient”
So side effects of drugs which arise due to interaction of the drug to molecules other than the target can be minimized by
making the drug more specific.

Ans.52
• Chloroquine is a safe antimalarial drug for pregnant women.
• Safe drugs for pregnant women :
 Chloroquine  Spiramycin  Methyldopa
www.targeteducare.com info@targeteducare.com
Helpline: 93 2233 6677 (Working days 11 AM to 6 PM)
TARGET EDUCARE PVT. LTD.
AIPGEE - 1 Explanations
 Hydralazine  Labetalol  Prazosin
 Zidovudine  Heparin  Clonidine
Note -
Propylthiouracil is not safe during pregnancy but it is the DOC for thyrotoxicosis in pregnancy.

Ans.53
The anticancer antibiotics are :
- Actinomycin – D (Dactinomycin) - Daunorubicin (Rubidomycin) - Mitomycin C
- Doxorubicin -Mitoxantrone - Mithramycin (plicamycin)
- Bleomvcins
• These anticancer, antibiotics obtained from micro-organisms and have prominent antitumour activity.
• Mechanism of action:
They intercalated between DNA strands and interfere with its template function.
Actinomycin ‘D’ inhibits DNA dependent RNA synthesis.
Bleomycin cause DNA breakage and free radical formation.
Doxo-and daunorubicin inhibit Topoisomerase I & II.
Mitomycin acts like alkylating agents.
Mitoxantrane binds to DNA to produce strand breakage and inhibits both DNA & RNA synthesis.

Ans.54
• Amongst the given optins pefloxacin causes phototoxicity.
Adverse effects
1. GI distrurbance – Nausea, Vomiting, Bad taste, anorexia (most common).
2. CNS – Dizziness, headache, restlessness, anxiety, insomnia, tremor, seizures → due to GABA antagonistic action.
3. Cartilage damage in weight bearing joints → Contraindicated in children.
4. Tendonitis and tendon rupture.
5. Skin/hypersensitivity – Rash, pruritis, urticaria.
6. Phototoxicity by lomefloxacin (maximum), sparfloxacin and pefloxacin.
7. Sparfloxacin, Gatifloxacin and moxifloxacin can prolong QT interval (Torsades de pointes).
Note – Following FQs have been withdrawn from the market because of their rare but potentially fatal side effects
(Goodman & Gilman 11th/e 1119)
• Temafloxacin → Immune hemolytic anemia • Grepafloxacin → Cardiotoxicity
• Trovafloxacin → Hepatotoxicity • Clinafloxacin → Phototoxicity

Ans.55
“The carbapenem imipenem has excellent activity against methicillin-sensitive S.aureus but not MRSA”.
MRSA is resistant to all β-lactam antibiotics because resistanec develops due to alteration in transpeptidase (penicillin binding
protein) on which all β-lactam antibiotics act.
Drugs for MRSA →
Vancomycin (DOC) Ciprofloxacin Quinopristin /dalfopristin Tigecyclin
TMP-SMX Linezolid Daptomycin
Minocyclin Levofloxacin Oritavaccin

Ans.56
• Antitubercular drugs
First line → Isoniazid, rifampin, ethambutol, streptomycin, pyrazinamide.
Second line → Thiacetazone, PAS, ethionamide, cycloserine, Kanamycin, amikacin, capreomycin, ciprofloxacin,
ofloxacin, clarithromycin, Azithromycin, rifabutin.
• Tobramycin is not an antitibercular aminoglycoside.

Ans.57
Causes of iron deficiency anemia

Diet
Malabsorption

www.targeteducare.com info@targeteducare.com
Helpline: 93 2233 6677 (Working days 11 AM to 6 PM)
TARGET EDUCARE PVT. LTD.
AIPGEE - 1 Explanations
Achlorhydria
Chronic, atrophic gastritis
Blood Loss - Gastric resection and bypass
• Gastrointestinal bleeding • Menstruation - Vagotomy
- Hemorrhoid • Frequent blood donations - Intestinal bypass operations
- Angiodysplasia of the colon • Erythrocyturia: bladder - Celiac disease (= nontropical sprue)
- Hereditary, hemorrhagic, teleangiectasia neoplasm - Tropical sprue
- Syndrome (Oster-Weber-Rendu • Hemoglobinuria Increased iron requirements
- Syndrome) - Paroxysmal, nocturnal - Pregnancy and lactation
- Peptic ulcer hemoglobinuria - Growth
- Nonsteroidal, anti-inflammatory drugs - Erythrocyte
- Oral anticoagulation fragmentation in
- Hiatal hernia prosthetic heart valves
- Meckel diverticulum • Factitious anemia
- Diverticulosis of the colon Intravenous and intra
- Intestinal polyps arterial drug abuse
- Carcinoma • Hemodialysis
- Inflammatory bowel disease • Nosocomial blood loss
- Hookworm: Nector Americanus. due to frequent The indices of the patient in question suggest a
- Ancylostoma duodenale venisection diagnosis of Microcytic Hypochromic anemia.
Shistosoma: S Mansoni,
The single best cause for his anemia is a
S. hematobium
Hookworm infection which causes an iron
- Trichuris
deficiency anemia due to persistant
gastrointestinal blood loss.

Ans.58
The most common mutation in beta thalassemia involves the intervening sequence 1 (IVS-1) or the intron-1

Mutations in Beta Thalassemia


• Most types of Beta thalassemia are caused by point mutations affecting one of few bases.
• A given mutation usually occurs within only one racial group and within a given racial group, five or six specific mutations
usually accounts for more than 90% of cases of Beta-thalassemia.
• The most common site of mutations in Beta thalassemia is the intervening sequence 1 (IVS-1) or Intron 1.

Ans.59
‘Bone marrow transplantation is the best therapy for the young patient with a fully histocompatible sibling donor.’

The curative treatment for young (< 20 year-old) patients with severe idiopathic aplastic anemia is allogenic bone marrow
transplantation if there is an available donor. Those with a compatible sibling donor should proceed to transplantation as soon as
possible.

Ans.60
‘In dentistry Surgery, General Anaesthesis is contraindicated in patients with Hemophilia’

Ans.61
Pernicious aneia is characterized by autoimmune destruction of parietal cells leading to hypochondria (↓ acid production). It is not
associated with peptic ulcer disease.
Aetiological Factors in peptic ulceration

Primary Aetiological Factors (selected)


• Endocrinal Disorders
- MEN
- Primary Hyperparathyroidism
- Zollinger Ellison SyndromeQ
- Cushing’s syndrome
• CirrhosisQ
• Chronic Renal Failure

www.targeteducare.com info@targeteducare.com
Helpline: 93 2233 6677 (Working days 11 AM to 6 PM)
TARGET EDUCARE PVT. LTD.
AIPGEE - 1 Explanations
• Chronic Pulmonary Disease
Primary Aetiological Factors (selected) • Chronic Paracreatic Disease
• H. pylori infection • Alpha l Antitrypsin deficiency
• NSAID’s and other drugs eg. Steroids
• Stree – Head injury / Burns
• Bile Reflus
• Smoking / Alcohol use
• Genetic Factors

Ans.62
Hydrochlorthiazide is not a preferred agent for the treatment of hypertension in diabetics as it causes unfavourable alteration in
glucose and lipid profile of the patient.
‘Hyperglycemia and hyperlipidemia have been reported with the used of diuretics as antihypertensives’ – KDT

Antihypertensive drugs in diabetics:


‘ACE inhibitors or angiotensin receptor blockers are the first line agents for management of hypertension in diabetics’ – Harrison 16th
/ 1479
Lisinopril & Trandalopvil are examples of ACE inhibitors while Losartan is an angiotensin receptor blocking agent.
These have no known adverse effects on glucose or lipid metabolism and minimize the development of diabetic nephropathy by
reducing renal vascular resistance and renal perfusion pressure.

What puzzles the situation:


In Diabetic patient with hypertension multiple agents are usually neede to achieve good blood pressure control. The average diabetic
patient will require at least three medications to achieve appropriate control. While ACE inhibitors/angiotensin receptor blockers will
form an essential component of the regimen. Thiazide diuretics may also be used as an additional agent. Thus diuretics though not
preferred agents for treatment of hypertension in diabetics, may well be used as conjunctive agents and are a possible treatment
modality. Nevertheless they are the single best answer to exclude amongst the options provided here.

Ans.63
IgM and IgG fix complement via the classical pathway
Frequently asked questions on immunoglobulins:
• Immunoglobulin to fix complements via classical pathway IgG & IgM (IgM > IgG)Q
• Immunoglobulin to fix complements via alternative pathway IgAQ & IgDQ
• Immunoglobulin with maximum serum conc. IgGQ
• Immunoglobulin with minimum serum conc. IgEQ
• Immunoglobulin that in heat labile IgEQ
• Immunoglobulin in primary immune response IgMQ
• Immunoglobulin in secondary immune response IgGQ
• Immunoglobulin with maximum molecular weight IgMQ
• Immunoglobulin present in milk IgAQ & IgGQ
• Immunoglobulin with maximum sedimentation coefficient IgMQ
• Immunoglobulin with shortest ½ life IgEQ
• Immunoglobulin in seromucinois glands IgG & IgAQ
• Immunoglobulin resp. for hypersensitive pneumonitis IgGQ
• Immunoglobulin mediating the prausnitz Kustner reaction IgEQ
• Homocytotropism is seen in which Ig IgEQ

Ans.64
The enzyme enolase requires Mg2+ or Mn2+ as a cofactor and it is inhibited by fluoride. That’s why for blood glucose estimation in the
laboratory, fluoride is added to the blood to prevent glycolysis by the cells, so that the blood glucose is correctly estimated.

Ans.65
Glycogen is the major storage carbohydrate in animals, corresponding to starch in plants; it is a branched polymer of alpha D-glucose.
It occurs mainly in liver (up to 6%) and muscle, where it rarely exceeds 1%. However, because of its greater mass, muscle contains
about three to four times as much glycogen as does liver. Muscle glycogen is a readily available source for glycolysis within the
muscle itself.

Ans.66
www.targeteducare.com info@targeteducare.com
Helpline: 93 2233 6677 (Working days 11 AM to 6 PM)
TARGET EDUCARE PVT. LTD.
AIPGEE - 1 Explanations
The steroid nucleus (ring structure) of the cholesterol cannot be degraded to CO2 and H2O. Cholesterol (50%) is converted to bile
acids, excreted in feces, serves as a precursor for the synthesis of steroid hormones, vitamin D, coprostanol and cholesatnol. The latter
two are the fecal sterols, besides cholesterol.

Ans.67
Fatty liver occurs during starvation and the feeding of high fat diets. The ability to secrete VLDL may also be impaired. In
uncontrolled diabetes mellitus, twin lamb disease, and ketosis in cattle, fatty infiltration is sufficiently severe to cause visible pallor
(fatty appearance) and enlargement of the liver with possible liver dysfunction.

Ans.68
Glutamate rapidly undergoes oxidative deamination, catalyzed by glutamate dehydrogenase (GDH) to liberate ammonia. It is the
major source of NH3 in kidney. This enzyme is unique in that it can utilize either NAD+ or NADP+ as coenzyme.
Ans.69
Heme biosynthesis occurs in most mammalian cells with the exception of mature erythrocytes, which do not contain mitochondria.
However, approximately 85% of heme synthesis occurs in erythroid precursor cells in the bone marrow and the majority of the
remainder in hepatocytes.

Ans.70
The main examples of defense proteins are snake venoms and immunoglobulins.

Ans.71
Ingestion of water is mainly controlled by the thirst center locate in the hypothalamus.

Ans.72
Na+ is the principal extracellular cation and K+ is the intracellular cation. This difference in the concentration is essential for the cell
survival which is maintained by Na+-K+ pump.

Ans.73
Aminopterin and amethopterin (also called as methotrexate) are structural analogues of folic acid. They competitively inhibit
dihydrofolate reductase and block the formation o THF. The biosynthesis of purines, thymine nucleotides and hence DNA is
impaired. This results in the blockage of cell proliferation. Aminopterin and methotrexate are successfully used in the treatment of
many cancers, including leukemia.

Ans.74
The symptoms of pellagra are commonly referred to as three D’s. The disease also progresses in that order dermatitis, diarrhoea,
dementia, and if not treated may rarely lead to death (4th D).

Ans.75
Vitamin B12 is also known as anti pernicious anemia vitamin. It is a unique vitamin, synthesized by only microorganisms and not by
animals and plants. It was the last vitamin to be discovered. Vitamin B12 is synthesized only by microorganisms (anaerobic
bacteria). Plants cannot synthesize, hence B12 is never found in plant foods. Animals obtain B12 either by eating foods, derived from
other animals or from the intestinal bacterial synthesis.

Ans.76
The enzyme commonly called “hexokinase” is designated “ATP: D-hexose-6-phosphotransferses E.C 2.7.1.1”. This identifies
hexokinase as a member of class-2 (transferases), subclass 7 (transfer of a Phosphoryl group), subclass 1 (alcohol is the Phosphoryl
acceptor). Finally, the term “hexose-6” indicates that the alcohol phosphorylated is that of carbon six of a hexose.

Ans.77
Immunoglobulins are the tumor markers for multiple myeloma.

Ans.78
Caries Susceptibility of different teeth:
In the pioneering studies of dental caries in Hagerstown, Maryland, carried out in 1937, the rank order of susceptibility of teeth to
caries was listed as follows:
• Mandibular first and second molars
• Maxillary first and second molars
• Mandibular second bicuspids, maxillary first and second bicuspids, maxillary central and lateral incisors
• Maxillary canines and mandibular first bicuspids

www.targeteducare.com info@targeteducare.com
Helpline: 93 2233 6677 (Working days 11 AM to 6 PM)
TARGET EDUCARE PVT. LTD.
AIPGEE - 1 Explanations
• Mandibular central and lateral incisors, mandibular canines

Ans.79
Another idea introduced the turn of the century by the well known dental histologist, J. LEON WILLIAMS, become the slogan of the
oral hygiene compaign of the following two decodes, “A clean tooth never decays”.

Ans.80
CLASSIFICATION OF INDICES:
I. Based upon the direction in which their scores fluctuate, indices are classified as:
1. Irreversible: Index that measures conditions that will not change. For Example: DMFT index for measuring dental caries.
2. Reversible: Index that measures conditions that can be changed. For example: Community Periodontal Index, Sulcular Bleeding
Index

II. Based upon the extent to which areas of oral cavity are measured, they are classified as:
1. Full Mouth Indices: These indices measure the entire periodontium or dentition. For example: Russel’s periodontal Index
2. Simplified Indices: These indices measure only a representative sample of dental apparatus. For example: Oral Hygiene Index-
simplified.

III. Based on the certain general categories, they are classified as:
1. Disease Index: The decay component of the DMFT index is the best example.
2. Symptom Index: The indices measuring gingival/sulcular bleeding is the best example.
3. Treatment Index: The filled component of the DMFT index is the best example.

IV. Based on the special categories as:


1. Simple Index: Index that measures the presence or absence of a condition. For example: Russel’s periodontal index.
2. Cumulative Index: Index that measures all the evidence of a condition, past and present. For example: DMF index for dental caries

Ans.81
YOUNG’S CLASSIFICATION OF ENAMEL FLUOROSIS
A classification for enamel fluorosis was developed by YOUNG, M. A., in 1973.
‘Young’ classified enamel fluorosis as follows:

Classification Criteria
Type A White areas less than 2 mm in diameter.
Type B White areas of, or greater than, 2 mm in diameter.
Type C Coloured (brown) areas of, or greater than 2 mm in diameter, irrespective of there being any white areas
Type D Coloured (brown) areas of, or greater than 2 mm in diameter, irrespective of there being white areas.
Type E Horizontal white lines irrespective of there being any white non-linear areas.
Type F Coloured (brown) or white areas or lines associated with pits or hypoplastic areas.

Ans.82
Lactobacillus Test:
Application:
• Lactobacillus levels are highly influenced by the intake of dietary carbohydrates, thus reflecting the amount of bacterial
substrate and indicating an acid environment within the oral cavity.

Ans.83
DEWAR TEST:
Principle:
This test is similar to the Fosdick Calcium Dissolution test. The only difference is that in Dewar Test, the final pH after 4 hours is
measured instead of the amount of calcium dissolved.
This procedure is not commonly used as it has not been adequately tested for clinical correlation.

Ans.84
TOOTH BRUSHING TECHNIQUES:
A number of tooth brushing techniques have achieved acceptance by the Dental Profession. Each technique has been designed to
achieve a definite goal. Hence, no one procedure can be described as the best. Depending on the individual cases, the techniques of
tooth brushing may have to be altered to achieve the maximum beneficial effect. These methods are classified broadly according to
Greene J.C (1966) AS:

www.targeteducare.com info@targeteducare.com
Helpline: 93 2233 6677 (Working days 11 AM to 6 PM)
TARGET EDUCARE PVT. LTD.
AIPGEE - 1 Explanations
The Roll technique: Modified Stillman / Rolling Stroke
The Vibratory technique: Stillman, Charters & Bass method
The Circular technique: Fones method
The Vertical technique: Leonard’s method
The Horizontal technique: “Scrub-Brush” method
The Physiological technique: Smiths method.

Ans.85
In recent years, there has been a steady increase in mental disorders. Alzheimer’s disease described as the “silent epidemic” of the
century is an important cause of morbidity and mortality.

Ans.86
Evaluation of a Screening Test:
a. Sensitivity (true positive) = a/ (a + c) X 100
b. Specificity (true negative) = d/ (b+ d) X 100
c. Predictive value of a positive test = a/ (a + b) X 100
d. Predictive value of a negative test = d/ (c + d) X 100
e. Percentage of false negatives = c/ (a + c) X 100
f. Percentage of false positives = b/ (b + d) X 100

Ans.91
Shrivastav committee, 1975
The Government of India in the Ministry of Health and Family planning had in November 1974 set up a ‘Group on Medical Education
and Support Manpower’ popularly known as the Shrivastav Committee:
(1) To devise a suitable curriculum for training a cadre of health assistants so that they can serve as a link between the qualified
medical practitioners and the multipurpose workers, thus forming an effective team to deliver health care, family welfare and
nutritional services to the people
(2) To suggest steps for improving the existing medical educational processes as to provide due emphasis on the problems
particularly relevant to national requirements,
(3) To make any other suggestions to realize the above objectives and matters incidental thereto.
The group submitted its report in April 1975. It recommended immediate action for :
(1) Creation of bands of para-professional and semi-professional health workers from within the community itself (e.g. school
teachers, postmasters, gram sevaks) to provide simple, promotive, preventive and curative health services needed by the
community;
(2) Establishment of 2 cadres of health workers, namely – multipurpose health workers and health assistants between the
community level workers and doctors at the PHC:
(3) Development of a ‘Referral Services Complex’ by establishing proper linkages between the PHC and higher level referral
and service centres, viz taluka/tehsil, district, regional and medical college hospitals, and
Screening test results Diagnosis Total
Diseased Not Diseased
Positive a (True Positive) b (False Positive) a+b
Negative c (False Negative) d (True Negative) c+d
Total a+c b+d a+ b+c+d
(4) Establishment of a Medical and Health Education Commission for planning and implementing the reforms needed in health
and medical education on the lines of the University Grants Commission.
The committee felt that by the end of the sixth Plan, one male and one female health worker should be available for every 5,000
population. Also, there should be one male and female health assistant for 2 male and 2 female health workers respectively. The
health assistants should be located at the subcentre, and not at the PHC.

Ans.92
Classification of amelogenesis imperfect according to Witkop (1989)
Type I Hypoplastic
IA Hypoplastic, pitted autosomal dominant
IB Hypoplastic, local autosomal dominant
IC Hypoplastic, pitted autosomal recessive
ID Hypoplastic, smooth, autosomal dominant
IE Hypoplastic, smooth X-linked dominant
www.targeteducare.com info@targeteducare.com
Helpline: 93 2233 6677 (Working days 11 AM to 6 PM)
TARGET EDUCARE PVT. LTD.
AIPGEE - 1 Explanations
IF Hypoplastic, rough autosomal dominant
IG Enamel agenesis, autosomal recessive
Type II Hypomaturation
IIA Hypomaturation, pigmented autosomal recessive
IIB Hypomaturation, X-linked recessive
IIC Snow-capped teeth, autosomal dominant
Type III Hypocalcified
IIIA Autosomal dominant
IIIB Autosomal recessive
Type IV Hypomaturation-hypoplastic with taurodontism
IVA Hypomaturation-hypoplastic with taurodontism, autosomal dominant
IVB Hypoplastic-hypomaturation with taurodontism, autosomal dominant

Ans.93
Central giant cell granuloma is essentially a destructive lesion, producing a radiolucent area with either a relatively smooth or a ragged
border, and sometimes showing faint trabeculae. Definite loculations are often present, particularly in larger lesions.

Ans.94
Kaposi’s sarcoma has a similar histopathologic appearance in all of its clinical subtypes. The early lesion (patch stage) is
characterized by a proliferation of small veins and capillaries around one or more preexisting dilated vessels.
More advanced lesions (plaque stage) are nodular and show increased numbers of small capillaries or dilated vascular channels
interspersed with proliferating sheets of sarcomatous or atypical spindle cells, often with large numbers of extravasated erythrocytes
and abundant hemosiderin deposition. In the nodular stage, all the histologic features are more prominent than plaque stage.

Ans.95
Hyaline body or Rushton body, often found in great numbers in the epithelium of apical periodontal or residual cysts. These hyaline
bodies are tiny linear or arc-shaped bodies, generally associated with the lining epithelium, that appear amorphous in structure,
cosinophilic in reaction and brittle in nature, since they evidence fracture in some cases. Their frequency of occurrence in cyst linings
ranges between 2.6 and 9.5 per cent of cysts, according to a review by Allison. The etiology, pathogenesis, and significance of these
structures are unknown.
Occasionally, the lumen may contain a great deal of cholesterol, and in rare instances, limited amounts of keratin are present.

Ans.96
Scrapings obtained from the base of the lesions are stained with Wright’s, and Giemsa stain. Pap stain demonstrates balloon cells,
multinucleated giant cells and intranuclear inclusions. Though cytological procedures give a quick results but it will not differentiate
between HSV and VZV more than that identification of giant cells requires experience.

Ans.98
It is characterized by a rapidly acquired adiposity about the upper portion of the body, mooning of the face, a tendency to become
round-shouldered and develop a ‘buffalo hump’ at the base of the neck, alteration in hair distribution, a dusky plethoric appearance
with formation of purple striae, muscular weakness, vascular hypertension, glycosuria not controlled by insulin, and albuminuria.

Ans.99
The pronounced anemia is of a hypochromic microcytic type, the red cells exhibiting a poikilocytosis and anisocytosis. These cells
are extremely pale, but in some instances appear as ‘target’ cells with a condensation of coloring matter in the center of the cell. The
presence of typical ‘safety-pin’ cells and of nor-moblasts or nucleated red blood cells in the circulating blood is also a characteristic
feature.

Ans.100
Oral lichen planus (OLP) is a common mucocutaneous disease. It was first described by Wilson in 1869 and is thought to affect 0.5-1
per cent of the world’s population. The condition can affect either the skin or mucosa or both. It can cause bilateral white striations,
papules, or plaques on the buccal mucosa, tongue, and gingivae. Erythema, erosions, and blisters may or may not be present. The
involvement of the oral mucous membrane is so frequent and accompanies or precedes the appearance of lesions on the skin and
genital mucous membrane.

Ans.101
Papillomas of the lips have been a striking feature in a number of these patients as well as papillomas of the buccal mucosa of
gingival. In addition, the teeth are commonly defective in size, shape or structure. Microdontia is a common finding as is enamel

www.targeteducare.com info@targeteducare.com
Helpline: 93 2233 6677 (Working days 11 AM to 6 PM)
TARGET EDUCARE PVT. LTD.
AIPGEE - 1 Explanations
hypoplasia. Cleft lips/cleft palate has also been described in several cases. Details of the disease have been discussed by Gorlin and
his associates.

Ans.102
The involved tooth is usually nonvital and may be slightly tender to percussion, and percussion may produce a dull sound instead of a
normal metallic sound because of the presence of granulation tissue around the root apex. Patients may complain about mild pain on
biting or chewing on solid food. In some cases, the tooth feels slightly elongated in its socket and may actually be so. The sensitivity
is due to hyperemia, edema, and inflammation of the apical periodontal ligament.

Ans.103
When lesions occur on the face, they normally appear on the cheeks, eyelids, and bridge of the nose, at times producing a butterfly-
shaped lesion that may resemble lupus erythematosus. If the eyelids are involved, they may become edematous and shut, thereby
resembling angioedema.

Ans.104
In some instances, there may be an associated osteogenic reaction, which exhibits a “sun ray” radiographic pattern that can be
mistaken for osteosarcoma. The tumor also can occur at other locations; the skull, mandible, brain, and epididymis or testis is the
most frequent extramaxillary sites. There is no apparent sex predilection.
High urinary levels of vanillylmandelic acid (VMA) often are found in patients with melanotic neuroectodermal tumor of infancy.

Ans.105
Radiographically, the most common feature is the presence of loose bodies in the joint. These consist of rounded, irregularly shaped,
and variably sized radiopaque structures in the region of the joint. Other features include irregularity of the joint space, widened joint
space, and irregularity of the condylar head.

Ans.106
Years Concepts Persons associated with
1895 Discovery of X-rays W. C. Roentgen

Ans.107
Characteristic radiation accounts for a very small part of X-rays produced in the dental X-ray machine and occurs only at 70 kVp and
above, because the binding energy of ‘K’ shell electron is approximately 70 kVp.

Ans.108
Type of film:
• Film speed: High speed films require less mAs in order to obtain a density change.
• Film latitude: It is measured as a range of exposures that can be recorded as distinguishable densities on a film.
• Radiographic noise: It is the appearance of uneven density of a uniformly exposed radiographic film. It is seen on a small area of
film as localized variations in density. The primary cause is radiographic mottle.
Screens: Use of screens require less mAs in order to obtain a density change.
Grids: The use of grids require more mAs in order to obtain a density change.
Amount of filtration used: Reduction in the amount of added filtration used will increase the number of photons reaching the film
and hence increase the density.
Fog: Film fog may result in an undesirable form of darkening of the film.

Ans.109
Composition
a. Base: Made of either a stiff sheet of cardboard or polyester plastic material (like the one used for the base of the radiographic
film). It is about 0.25 mm thick. The base is the supporting component of the screen.
b. Reflecting layer: This is a thin layer of white material (i.e. magnesium oxide or titanium dioxide) between the base and the
luminescent layer. It serves to redirect to the film a large fraction of the emitted visible light which is moving away from the film
and which would therefore otherwise be lost. Thus, it increases the sensitivity but some degree of unsharpness is created because
of divergence of light reflected back to the film.
c. Phosphor layer: This layer consists of light sensitive phosphor crystals suspended in a plastic material.
When these crystals are struck by photons they fluoresce, that is, they emit visible light photons that expose the X-ray film. Even
a single X-ray photon absorbed in an intensifying screen generates many light photons leading to increased film exposure. The
speed of the screens increases as the crystal size increases but the overall image quality may be degraded.
Types of phosphor used in dental screens are:

www.targeteducare.com info@targeteducare.com
Helpline: 93 2233 6677 (Working days 11 AM to 6 PM)
TARGET EDUCARE PVT. LTD.
AIPGEE - 1 Explanations
i. Crystalline Calcium Tungstate, that fluoresces in the blue portion of the spectrum. Conventional screens (Kodak X-Omatic
Regular screens) are used with blue sensitive films.
ii. Rare earth intensifying screens using terbium activated gadolinium oxysulphide and thelium activated lanthanum
oxybromide, which fluoresce in the green portion of the spectrum.

The newer rare earth screens have phosphors that emit green light. The term ‘rare earth’ is used because it is difficult and
expensive to separate these elements from earth and from each other and not because these elements are rare. Rare earth
intensifying screens are four times more efficient than calcium screens and are considered faster, and thus less exposure is
required when these rare earth screens are used. Rare earth screens (Kodak Lanex Regular and Medium screens) are designed for
use with green sensitive films.
d. Coat: This layer protects the phosphor layer from mechanical insult such as abrasion, scratching, etc. it is important to keep the
intensifying screens clean, because any debris, spots which are opaque to visible light or scratches will result in light (under
exposed) spots on the resultant radiograph.

Ans.110
The basic principles of projection geometry (shadow casting) are:
1. The focal spot (source of radiation) should be as small as possible
2. The focal spot-object distance should e as long as possible
3. The object-film distance should be as small as possible
4. The long axis of the object and the film planes should be parallel
5. The X-ray beam should strike the object and the film planes at right angles
6. There should be no movement of the tube, film or patient during exposure. (Given by Mason and Lincoln)

Ans.111
Summary of Comparison between Short cone and Long cone Technique
Short cone Long cone
1. Diffusion and distortion of the image 1. Sharp details of the image obtained
2. Increased chances of elongation of shortening of the image 2. Image obtained is of the same size and shape as the object
3. Distorted image of the teeth due to oblique exposure and 3. Image of the teeth nearly anatomically accurate, from use of
bending of the film right angle exposure and flat surface of the film
4. Shadows of the alveolar bone tend to fill the interproximal 4. Alveolar crest seen in true relationship to the teeth
spaces 5. Less vertical angulation, thus similar buccal and lingual parts
5. More vertical angulation of the teeth appear closed to each other in the radiograph, and
more tooth area underneath restorations is revealed.
6. Superimposition of the shadow of the zygomatic arch on the 6. Less vertical angulation in the maxillary molar region avoids
teeth the shadow of the zygomatic arch and the teeth apices and
maxillary sinus are better seen
7. Easier technique to maneuver and requires less space 7. Needs a larger working space
8. More effective when the palate is shallow, children with 8. In a similar situation apices of the teeth may be cut off
adult size teeth but underdeveloped jaws 9. This is not possible in the long cone technique
9. In rare cases when the teeth are longer than film, the entire
tooth may be seen, by over angulating the vertical angulation 10. The PID, helps reduce such errors
10. Cone cutting is a common error especially, in the maxillary
third molar area 11. Use of film holding device prevents such an error.
11. Curved film due to incorrect finger pressure

Ans.112
Typical radiographic description of various oral lesions as seen on the radiograph
Egg shell appearance Ameloblastoma
Multilocular cyst

Ans.113
Immunohistochemistry, gel electrophoresis, and immunoblot techniques have made identification of the characteristic pattern of
cytokeratins possible in each epithelial type. The keratin proteins are composed of different polypeptide subunits characterized by
their isoelectric points and molecular weights. They are numbered in a sequence contrary to their molecular weight.

www.targeteducare.com info@targeteducare.com
Helpline: 93 2233 6677 (Working days 11 AM to 6 PM)
TARGET EDUCARE PVT. LTD.
AIPGEE - 1 Explanations
Ans.114
Within 2 weeks, a nearly mature microbiota is established in the gut of the newborn. After weaning (>2 years), the entire human
microbial flora is formed by a complex collection of approximately 10 microorganisms consisting of more than 400 different types of
bacteria.

Ans.115
The transition from gram-positive to gram-negative microorganisms observed in the structural development of dental plaque is
paralleled by a physiologic transition in the developing plaque.

Ans.116
Many periodontal pathogens stimulate the production of interleukin-8 (IL-8), a proinflammatory chemokine that provides a signal for
the recruitment of neutrophils (PMNs) to local site (see later discussion). P. gingivalis is able to inhibit the production of IL-8 by
epithelial cells, which may provide the microorganism with an advantage in evading PMN-mediated killing.

Ans.117
The initial biologic processes and bone remodeling needed to complete the first remodeling cycle of bone around an implant require
an estimated 4 months in humans.

Ans.118
Neutropenia is a blood disorder that results in low levels of circulating neutrophils. An individual with an absolute neutrophil count
(ANC) of less than 1500 cells per microliter is considered to beneutropenic.

Ans.119
Prominent periodontal pathogen such as P. gingivalis, T. forsythia, F. nucleatum, P. micros, E. corrodens, and Streptococcus
intermedius have been found to be elevated in patients who do not respond to treatment.

Ans.120
The chronic gingivitis, which occurs 2 to 3 weeks after the beginning of plaque accumulation, the blood vessels become engorged and
congested, venous return is impaired, and the blood flow becomes sluggish.

Ans.121
Wasting Disease of the Teeth
Wasting is defined as any gradual loss of tooth, substance characterized by the formation of smooth, polished surfaces, without regard
to the possible mechanism of this loss. The forms of wasting are erosion, abrasion, attrition, and abfraction.

Ans.122
Exostoses are outgrowths of bone of varied size and shape. Palatal exostoses have been found in 40% of human skulls.

Ans.123
Phases of Periodontal Therapy
Surgical phase (Phase II Therapy)
Periodontal therapy, including placement of implants
Endodontic therapy

Ans.124
The maintenance phase of periodontal treatment starts immediately after the completion of Phase I therapy.

Ans.125
Internal bevel incision is the initial incision in the reflection of periodontal flap. It is also known as reverse bevel incision.

Ans.126
It can lead to bone necrosis if electrode touches the bone, can also cause cementum burns.

Ans.127
Lymphatic Drainage of the Face
The face has three lymphatic territories:
(a) The upper territory, including the greater part of the forehead, the lateral halves of the eyelids, the conjunctiva, the lateral part
of the cheek and the parotid area, drains into the preauricular parotid nodes.

www.targeteducare.com info@targeteducare.com
Helpline: 93 2233 6677 (Working days 11 AM to 6 PM)
TARGET EDUCARE PVT. LTD.
AIPGEE - 1 Explanations
(b) The middle territory, including a strip over the median part of the forehead, the external nose, the upper lip, the lateral part of
the lower lip, the medial halves of the eyelids, the medial part of the cheek, and the greater part of the lower jaw, drains into
the submandibular nodes.
(c) The lower territory, including the central part of the lower lip and the chin, drains into the submental nodes.

Ans.128
The temporal branches cross the zygomatic arch and supply:
(a) The auricularis anterior.
(b) The auricularis superior.
(c) The intrinsic muscles on the lateral side of the ear.
(d) The frontalis.
(e) The orbicularis oculi.
(f) The corrugator supercilii.

Ans.129
Branches of First Part of the Maxillary Artery
1. The deep auricular artery supplies the external acoustic meatus, the tympanic membrane and the temporomandibular joint.
2. The anterior tympanic branch supplies the middle ear including the medial surface of the tympanic membrane.
3. The accessory meningeal artery enters the cranial cavity through the foramen ovale. Apart from the meninges it supplies
structures in the infratemporal fossa.
4. The inferior alveolar artery runs downwards and forwards medial to the ramus of the mandible to reach the mandibular foramen.
Passing through this foramen the artery enters the mandibular canal (within the body of the mandible) in which it runs downwards
and then forwards.
Before entering the mandibular canal the artery gives off a lingual branch to the tongue; and a mylohyoid branch that descends in
the mylohyoid groove (on the medial aspect of the mandible) and runs forwards above the mylohyoid muscle.
Within the mandibular canal the artery gives branches to the mandible and to the roots of the each tooth attached to the bone. It
also gives off a mental branch that passes through the mental foramen to supply the chin.

Ans.130
Submandibular Ganglion
This is a parasympathetic peripheral ganglion. It is a relay station for secretomotor fibres to the submandibular and sublingual
salivary glands. Topographically, it is related to the lingual nerve, but functionally, it is connected to the chorda tympani branch of the
facial nerve.
The fusiform ganglion lies on the hyoglossus muscle just above the deep part of the submandibular salivary gland, suspended from the
lingual nerve by two roots.
Conenctios and Branches
1. The motor or parasympathetic fibres pass from the lingual nerve to the ganglion through the posterior root. These are
preganglionic fibres that arise in the superior salovatory nucleus and pass through the facial nerve, the chorda tympani and
the lingual nerve to reach the ganglion.
The fibres relay in the ganglion: Postganglionic fibres for the submandibular gland reach the gland through five or six
branches from the ganglion.
Postganglionic fibres for the sublingual and anterior lingual glands re-enter the lingual nerve through the anterior root and
travel to the gland thought the distal part of the lingual nerve.
2. The sympathetic fibres are derived from the plexus aroud the facial artery. It contains postganglionic fibres arising in the
superior cervical ganglion. They pass through submandibular ganglion without relay, and supply vasomotor fibres to the
submandibular and sublingual glands.
3. Sensory fibres reach ganglion through the lingual nerve.

Ans.131
The root portion of the tooth is firmly fixed in the bony process of the jaw, so that each tooth is held in its position relative to the
others in the dental arch. That portion of the jaw serving as support for the tooth is called the alveolar process. The bone of the tooth
socket is called the albeolus (plural, alveoli).
The crown portion is never covered by bone tissue after fully erupted, but it is partly covered at the cervical third in young adults by
soft tissue of the mouth known as the gingiva or gingival tissue, or gums. In some persons, all of the enamel and often some cervical
cementum may not be covered by the gingiva.

Ans.132
Masseter Muscle
The masseter muscle extends from the zygomatic arch to the ramus and body of the mandible. The insertion of this muscle is broad,
extending from the region of the second molar on the lateral surface of the mandible to the posterior lateral surface of the ramus. The
www.targeteducare.com info@targeteducare.com
Helpline: 93 2233 6677 (Working days 11 AM to 6 PM)
TARGET EDUCARE PVT. LTD.
AIPGEE - 1 Explanations
masseter muscle is covered partly by the platysma muscle and by the risorius muscle. The platysma is activated during firm clenching
in some individuals and, having some insertion in the orbicular muscle (orbicularis oris), is sometimes active in facial expression. The
risorius is affected by emotion and is active in facial expression.
The superficial part of the masseter muscle is separated distinctly only from the deeper layer of the muscle at the posterior upper part
of the muscle. The masseter muscle is covered partly and to a variable degree with the parotid gland tissue. The center of the lower
third of the masseter muscle is about 2 to 3 cm from the anterior border of the sternocleidomastoid muscle, which contracts during
clenching in some individuals. The masseter muscle is active during forceful jaw closing and may assist in protrusion of the
mandible. The masseter muscle is innervated by the fifth nerve (masseter nerve). The zygomaticomandibular muscle (deep masseter
muscle) inserts at the coronoid process and originates on the inner surface of the zygomatic arch. It may be an antagonist to the
posterior temporalis and a synergist for the lateral pterygoid muscle.

Ans.133
Fibers
The collagen fibers in the pulp exhibit typical cross striations at 64 nm (640 Å) and range in length from 10 to 100 nm or more. The
main type of collagen fiber in the pulp is type I. Type III collagen is also present. Bundles of these fibers appear throughout the pulp.
In very young pulp fine fibers ranging in diameter from 10 to 12 nm (100 to 120 Å) have been observed. These fine fibers are called
fibrillin. Their significance is unknown. Pulp collagen fibers do not contribute to dentin matrix production, which is the function of
the odontoblast. After root completion the pulp matures and bundles of collagen fibers increase in number. They may appear
scattered throughout the coronal or radicular pulp, or they may appear in bundles. These are termed diffuse or bundle collagen
depending on their appearance, and their presence may relate to environmental trauma. Fiber bundles are most prevalent in the root
canals, especially near the apical region.

Ans.134
• Bacterial Culture and Antibiotic Sensitivity tests
Cultures can be obtained from the throat, spuntum, draining pus, urine or stools. Cultures from the oral cavity can be obtained either
by gathering exudative material or by aspiration with a needle and syringe or by use of a swab. A throat or pus culture is obtained by
collecting the sample on an autoclaved swab stick. A transport medium must be used when the sample cannot be inoculated
immediately onto a primary culture medium. For smear and culture in suspected cases of tuberculosis, the sample is sent in
normal saline.
• FNAC
Fine Needle Aspiration Cytology uses a fine needle (23-26G) to aspirate the contents of the lesion. A smear is prepared, stained and
studied. This test may not be significant at times.

Ans.135
Two Ways of Bone Removal
a. High speed, high toque handpiece and but technique
b. Chisel and mallet technique.
Bur technique round bur or a straight no. 703 fissure bur is used. Either of these burs can be used for bone removal or for sectioning
of a tooth. Burs should be always used along with copious saline irrigation to avoid thermal trauma to the bone.

Ans.136
A deep level of G.A. is enhanced by increasing alveolar concentration of L.A.
Minimum alveolar concentration (MAC) of an anesthetic agent is to produce lack of reflex response to skin incision in 50% of
patients. Volatile anesthetic agents are rapidly removed from the body via lungs and little evidence of metabolism can be
demonstrated following short exposure.

Ans.137
Ketamine is a phencyclidine derivative. It is a nonbarbiturate hypnotic, which produces dissociative type of anaesthesia. It produces
sedation, amnesia and intense analgesia. It has a very high margin of safety. It has good tissue compatibility (no irritation of veins).
Since the intraoral muscles, particularly, those of the tongue, do not become relaxed under the effect of ketamine, the airway remains
unobstructed. This unique characteristic is ideal for dentistry. The addition of sedative premedication increases the incidence of
aspiration especially, when ketamine is used in anaesthetic doses (1-2 mg/kg IV or 8 to 10 mg/kg IM).

Ans.138
Apnoea may be due to the following:
a. Respiratory obstruction.
b. Breath holding in light anaesthesia.
c. Deep anaesthesia.
d. Severe hypoxia.

www.targeteducare.com info@targeteducare.com
Helpline: 93 2233 6677 (Working days 11 AM to 6 PM)
TARGET EDUCARE PVT. LTD.
AIPGEE - 1 Explanations
e. Grave cardiovascular depression requiring resuscitation.
f. Fainting-fear, anxiety and occasionally pain in the patients presenting for outpatient dental anaesthesia, can cause significant
autonomic nervous system overactivity.

Ans.139
The hypertonic saline or sclerosing solution is used for TMJ subluxation is paracapsular.

Ans.140
- Bilateral submandibular incisions and if required a midline submental incision 1 cm below the inferior border of mandible are
sufficient to drain the involved spaces.
- Following aseptic technique and adequate local anaesthesia, an incision is made, which divides skin, the superficial fascia,
platysma, investing layer of deep fascia, geniohyoid and finally mylohyoid muscles.
- To be effective, it is essential to divide deep fascia and the mylohyoid muscle, then only it will drain sublingual space. A through
and through drain is placed to relieve the intense pressure of the oedematous tissues on the airway. Sublingual space can be
separately drained by intraoral approach if required and if oral opening permits it. Surgical drain should be secured by suturing to
the margins of the incision.

Ans.141
Whenever antral puncture is to be carried out; the puncture into the sinus cavity should be made through the middle meatus in
children; and in the inferior meatus in adults. The antral puncture or intranasal antrostomy, in the inferior meatus then becomes a point
of dependent drainage for maxillary sinus. The floor of maxillary sinus is about 1.25 cm below the floor of the nose.

Ans.142
Submandibular incision is use to access the mandibular ramus, angle and posterior body. Patient is prepared and draped in routine
surgical manner. Important landmarks-the corner of the mouth and the eyeglobe must be visible. Head of the patient is turned
sideways. The incision is marked 2 cm below the inferior border of the mandible to avoid damage to the marginal mandibular branch
of the facial nerve. Ideally the incision is placed in a relaxed skin tension line (the Langer’s lines)

Ans.143
Advantages of Rigid Fixation:
i. Rigid or stable fixation.
ii. Obviates the need for immobilization of the mandible.
iii. Early return to home and work.
iv. Soft diet can be taken.
v. Maintenance of oral hygiene.
vi. Useful in mentally challenged, physically handicapped patients.
vii. Maintenance of airway in multiple fractures.
The types of Rigid Fixation includes-
a. Simple noncompression bone plates
b. Mini bone plates
c. Compression plates
d. Reconstruction plates
Whereas, Transosseous Wiring (Intraosseous Wiring) is a direct wiring across the fracture line is an age old and effective method of
fixation of jaw bone fractures.
Transosseous wiring can be done through intraoral or extraoral approach. In principle, holes are drilled in the bony fragments on
either side of the fracture line, after which a length of 26 gauge stainless steel wire is passed through the holes and across the fracture.
After this initial preparation, the fracture must be reduced independently with the teeth in occlusion before the free ends of the wire
are lightened and twisted. The twisted ends are cut short and tucked into the nearest drill hole. The single strand wire fixation in this
horizontal manner is the simplest form of fixation with intraosseous wiring. But this can be modified in various ways depending on
the need.

Ans.144
Caldwell’s Technique
• Entire lingual mucoperiosteal flap is reflected from molar to molar region.
• Mylohyoid ridge is reduced / removed along with the reduction of genial tubercle.
• Mylohyoid muscle and superficial fibres of genioglossus muscles are pushed inferiorly.
• Rubber tubing placed in the lingual vestibule and the flap is held in position at the vestibular depth, by sutures passed through the
skin extraorally, at the inferior border of the mandible.

www.targeteducare.com info@targeteducare.com
Helpline: 93 2233 6677 (Working days 11 AM to 6 PM)
TARGET EDUCARE PVT. LTD.
AIPGEE - 1 Explanations
Ans.145
The cyst itself is frequently symptomless and may be discovered, when periapical radiographs are taken of teeth with nonvital pulps.
Slowly enlarging swellings are often complained of. Radicular cysts at times attain a large size. Pain may be a significant chief
complaint, in the presence of suppuration. Initially, the enlargement is bony hard, as the cyst increases in size; the covering bone
becomes thin and exhibits springiness due to fluctuation. In the maxilla, buccal and palatal or only palatal expansion due to the lateral
incisor or a palatal root will be noted. In the mandible, lingual expansion is very rare. The mucosa overlying the cysts expansion, as
with the other cysts, is at first of normal colour, then it may become conspicuous because then it may become conspicuous because of
the presence of dilated blood vessels and finally it will take on a profound dark bluish tingue, in case of large cysts. An intraoral sinus
tract may be identified with discharging pus or brownish fluid, when the cyst is infected. The involved tooth / teeth will be found to
be nonvital, discoloured, fractured or with heavy restorations or a failed root canal. They may be sensitive to percussion or
hypermobile, or displaced. It may involve deciduous or the permanent dentition. Temporary paraesthesia or anaesthesia of the
regional nerve distribution may be evident as with other cysts when infection is present. Pathologic fracture may be the form of
presentation in the mandible, as with other large cysts.

Ans.146
Whenever premolar extractions are carried out for relieving the crowding, the total extraction space should not be utilized. At least
3 to 4 mm space should be left for making the segmental osteotomy cuts, without jeopardizing the periodontal status of the adjacent
teeth. Judicious inter-proximal stripping can be carried out wherever possible, so that the entire premolar space can be utilized for set
back of the anterior segment.
Periapical X-rays of the planned osteotomy region should be taken and if roots are convergent, they are made divergent. The equal
amount of space should be created between the crowns as well as roots of the teeth adjacent to the osteotomy sites.

Ans.147
Mandibular nerve is the nerve of first (mandibular) branchial arch. The first ach gibes rise to the following:
• Precursor of mandible (Meckel’s cartilage)
• Spine of sphenoid
• Sphenomandibular ligament
• Muscles of mastication.
It is the largest branch of trigeminal nerve.
It transmits sensory fibres from the skin over the mandible, side of the cheek and temple, the oral cavity and its contents,
external ear, tympanic membrane and TM joint. It also supplies the meninges of cranial vault.
It is motor supply to the muscles derived from first branchial arch:
• Temporalis and masseter
• Medial and lateral pterygoid
• Mylohyoid and anterior belly of digastrics
• Tensor tympani and tensor palati.
- Note the four groups of two-two tensors, two pterygoids, and two big chewing muscles and two in floor of mouth.
- Some of its distal branches also convey parasympathic secretomotor fibres to salivary glands and taste fibers from
anterior portion of tongue.

Ans.148
• Class I arches are most common and class IV is least common.
• Class I and class II, long span class III and IV partial dentures are tooth-tissue supported prostheses.
• Short span class III and IV are tooth supported partial dentures.

Ans.149
• The retentive undercut will be present only in relation to a given path of insertion. The retentive undercut is absent in conditions
where the direction of dislodgement of the clasp arm is similar to the direction along which the clasp arm was inserted. Hence it
is important to maintain a single path of insertion that does not coincide with the path of displacement.
• Path of displacement is any path of movement of the clasp without resistance. Hence, there will be no height of contour
obstructing the movement of the retentive arm along any path of displacement. It the path of insertion and displacement of the
denture are parallel to one another it simply means that there is no retentive undercut present along the path of insertion.

Ans.150
After tripoding the primary cast, three additional reference points are marked on the cast using the same procedure described to mark
the first three tripod marks. The commonly used additional reference points are:
• Distal marginal ridge of the first premolar.
• Incisal edge of lateral incisor.
• Lingual cusp tip of the first premolar on the opposite side (opposite to the side where the other two points were marked.
www.targeteducare.com info@targeteducare.com
Helpline: 93 2233 6677 (Working days 11 AM to 6 PM)
TARGET EDUCARE PVT. LTD.
AIPGEE - 1 Explanations
Ans.151
• Surgical Splints are used to protect post-operative surgical sites in the oral cavity to improve the healing. They are most
commonly used on the maxillary arch and the lingual side of mandibular ridge where tori and exostoses are most likely to occur.
• Clinical procedure – Impression is made with irreversible hydrocolloid.

Ans.152
The Buccal Frenum separates the labial and buccal vestibule. It has attachments of the following muscles,
Levator anguli oris - Attaches beneath the frenum
Orbicularis oris - Pulls the fernum in a forward direction
Buccinators - Pulls the frenum in the backward direction.
These muscles influence the position of the buccal frenum hence it needs greater (wider and relatively shallower) clearance on the
buccal flange of the denture.

Ans.153
Anthropometric cephalic index - The transverse circumference of the head is measured using a measuring tape at the level of the
forehead. The width of the upper central incisor can be derived from this measurement. Sears called this formula as the
anthropometric cephalic index.

Width of the upper central incisor = Circumference of the head


13

Ans.154
Lingualized Occlusion – was first proposed by Alfred Gysi in 1927. This type of occlusion involves the use of a large upper palatal
cusp against a wide lower central fossa. In this scheme, the buccal cusps of the upper and lower teeth do not contact each other.
Clough reported that 67% of the patients preferred lingualized occlusion due to its superior chewing efficiency.

Ans.155
Advantages and Disadvantages of Different Margin Designs
Advantages Disadvantages Indications
Slopped Bulk of material, advantages of Less conservative of tooth structure Facial margins of metal-
shoulder bevel ceramic crowns

Ans.156
• If the distance between the finish line and the alveolar crest (combined width of epithelial and connective tissue attachments) is
less than 2.0 mm, the restoration may lead to gingival inflammation, loss of alveolar crest and pocket formation. A crown
lengthening procedure should be done to move the alveolar crest to a location about 3.0 mm away from the finish line to preserve
the periodontal health.
• The finish line should be at least 3 mm away from the alveolar crest to preserve the periodontium.

Ans.157
Full Veneer Crowns – These retainers cover all the five surfaces of the abutment. They are fabricated like a cap and are usually
indicated for extensively damaged teeth. They are most retentive and ideal retainers because their design can resist masticatory forces
in all directions.

Ans.158
Replacement of a Single Missing Canine – In cases of a single missing canine, where a cantilever FPD is planned, support should be
taken from both central and lateral incisor.

Ans.159
Features Their functions
• Chamfer finish line: • Marginal integrity and Periodontal preservation

Ans.160
The major Advantages of Fixed Fixed Partial Dentures include:
• Easy to fabricate
• Economical design
• Strong
• Easy to maintain
www.targeteducare.com info@targeteducare.com
Helpline: 93 2233 6677 (Working days 11 AM to 6 PM)
TARGET EDUCARE PVT. LTD.
AIPGEE - 1 Explanations
• Robust design provides maximum retention and strength
• Helps to splint mobile abutments
• Can be used for long bridges along with periodontally weak abutments.
Disadvantages
• Since the connectors are rigid, unwanted stress and lever forces are directly transferred to the abutment producing considerable
damage.
• Requires excessive tooth preparation to achieve a single path of placement.
• Difficult to cement on multiple abutments
• Contraindicated for pier abutments.
• A fixed partial denture constructed over these abutments will produce rotational forces around the pier (2) abutment.

Ans.161
Depending on the Materials Used, the implants can be classified into Metallic implants (Titanium, Titanium alloy, Cobalt Chromium
Molybdenum alloy) and Non-metalic implants (Ceramics, Carbon).

Ans.162
Contact angle of Porcelien is less, hence better wetting.

Ans.163
If the concentration of tin is less than 26%, the beta (β) phase, which is a solid solution of sliver and tin, forms. In one product, 5% tin
is replaced by 5 % indium, whereas another product contains less than 1% palladium. Adding this small amount of palladium
enhances the mechanical properties and corrosion resistance. The replacement of silver by an equal amount of copper produces a
copper tin compound(Cu3Sn).

Ans.164
A resin composite is composed of four major components: organic polymer matrix, inorganic filler particles, coupling agent and the
initiator accelerator system. The organic polymer matrix in most commercial composites today is either an aromatic or urethane
diacrylate oligomer. Oligomers are viscous liquids, the viscosity of which is reduced to a useful clinical level by the addition of a
diluents monomer.

Ans.165
Most modern bonding agents bond best to a moist tooth. If dentin is overdried, it is best to rehydrate it by applying a moist cotton
pellet for 15 seconds before applying the primer of the bonding agent.

Ans.166
Odontoblastic processes sometimes cross DEJ into enamel. When their ends are thickened, they may serve as pain receptors. These are
called enamel spindles.

Ans.167
The deepest zone is known as the dark zone because it does not transmit polarized light. This light blockage is caused by the presence
of many tiny pores too small to absorb quinoline. These smaller air-filled or vapor-filled pores make the region opaque. The total
pore volume is 2% to 4 % . There is some speculation that the dark zone is not really a stage in the sequence of the breakdown of
enamel; rather, the dark zone may be formed by deposition of ions into an area previously containing only large pores. Caries is an
episodic disease with alternating phases of demineralization and remineralization.

Ans.168
Linear Coefficients of Thermal Expansion

Biomaterials/ structures LCTE (ppm/0C)

Aluminous dental porcelain 4

Alumina 6.5-8

In-Ceram 8-10

Cp-litanium 8-9

www.targeteducare.com info@targeteducare.com
Helpline: 93 2233 6677 (Working days 11 AM to 6 PM)
TARGET EDUCARE PVT. LTD.
AIPGEE - 1 Explanations
Traditional dental cements 8-10

Tooth structure 9-11

Stainless steel 11

PFM ceramics 14

PRM alloys 14

Gold foil 14-15

Gold casting alloys 16-18

Co-cr alloys 18-20

Hybrid glass-ionomers 20-25

Dental amalgam 25

Packable composites 28-35

Anterior and flowable composites 35-50

Composite cements 40

PMMA direct-filling resins 72-83

Dental wax 260-600

Ans.169
It can be differentiated from primary dentin by the sharp bending of the tubules producing a line of demarcation.

Ans.170
• Chronic Periradicular diseases with areas of rarefaction
Chronic alveolar abscess
Granuloma
Cyst
• Chronic Periradicular disease with area of condensation
Condensing osteitis

Ans.171
K – File (Kerr manufacturing company)
- Has got the name from its manufacturing company.
- Manufactured from stainless steel square blank.

Ans.172
Sotokawa’s classification of instrument damage is:
1. Type I : Bent instrument
2. Type II : Stretching or straightening of twist contour.
3. Type III : Peeling-off metal at blade edges.
4. Type IV : Partial clockwise twist.
5. Type V : Cracking along axis.
6. Type VI : Full fracture

Ans.173
Local Contraindications-

www.targeteducare.com info@targeteducare.com
Helpline: 93 2233 6677 (Working days 11 AM to 6 PM)
TARGET EDUCARE PVT. LTD.
AIPGEE - 1 Explanations
1. Root Fractures : Vertical fractures have a poor prognosis.
2. Destruction of the periapical tissues involving more than 1/3rd of the length of the root.
3. Obstructed root canal of a pulpless tooth with a radiolucent area, i.e. curved root a tortuous canal, secondary dentin, a pulpstone
that cannot be removed or by passed, a calcified or partially calcified canal a malformed tooth or a broken instrument.
4. Bizarre anatomy, i.e. incomplete development of root apex and death of the pulp.
5. Accidental or pathologic perforations of the root surface.
6. Insufficient periodontal support.
7. Massive internal or external resorption.
8. When there is persistent excessive periapical exudates that cannot be controlled prior to filling the root canal or when negative
cultures cannot be obtained.
9. In cases of treatment, when a foreign body such as a fragment of gutta-percha or of root canal filling material, lies in the
periapical tissues of radiolucent teeth.
10. When on acute infection in a preciously treated and filled pulpless tooth has occurred, treatment is indicated after the acute
symptoms are controlled.

Ans.174
Carve
- Carving is for shaping.
- Reamers are best for carving.
- Instrument should not be pressed apically but simply touches the dentin and shape on withdraw.

Ans.175
• High temperature Injection Moulded Method
• Thermoplasticized GP is heated to 700C – 1600C depending on the method or material used.

Ans.176
• The alveolar arches of an infant are called gumpads, which are firm and pink structures with a definite from.
• Each gumpad is divided into ten segments by transverse grooves. The groove between the deciduous canine and first molar
segments are prominent and called the lateral sulci.
• The upper gumpad is horseshoe shaped and shows:
- Gingival groove is the groove separating the gumpad from the palate.
- Dental groove is originates in the incisive papilla region and extends backwards to touch the gingival groove in the canine
region and then laterally to end in the molar region.
- Lateral sulcus is a deepened groove separating the canine and deciduous first molar segments.

Ans.177
• Caufield (1993) monitored oral cavity levels from birth up to 5 years. He noted the initial acquisition mutans S. and designated
the time period as ‘window of infectivity’. As the teeth (primary teeth) erupt into the oral cavity they provide a virgin habitat
which enables MS to colonize the oral cavity avoiding competition with other indigenous bacteria. Thus in the window period in
deciduous teeth the MS is established by 7-31 months of age and may have difficulty in establishing later because it would need
to compete with other indigenous bacteria.
• Krass et al (1967), Edrman et al (1975) reported that at 2-6 yr of age the child is less susceptible to acquiring MS. The ‘Second
Window of Infectivity’ is present in permanent dentition between 6-12 years of age. (Klock and Kroske (1977). 90% of
teenagers have MS colonization while others found only 3% of adults (mothers).

Ans.178
Facial injuries prevalence in physical child abuse
Site Percentage
Scalp 79
Neck 59
Forehead 52
Cheek 49
Lower jaw 48
Upper lip 45

Ans.179
The Mucocele is an extravasation type of cyst (or can also be a retention cyst). The cause commonly attributed is trauma to minor
salivary glands with the mucus/fluid spreading into adjacent tissue which gets covered by a fibrous lining. Credence is given to this

www.targeteducare.com info@targeteducare.com
Helpline: 93 2233 6677 (Working days 11 AM to 6 PM)
TARGET EDUCARE PVT. LTD.
AIPGEE - 1 Explanations
theory by the fact that they most frequently occur on the lower lip area. Plain puncturing of the lesion is associated with a recurrence
and thus enucleation of the lesion along with removal of the adjacent minor salivary glands is the treatment of choice.

Ans.180
Indication for Serial Extraction
1. Class I with anterior crowding (space discrepancy 10 mm or more)
2. Lingual eruption of the lateral incisors.
3. Midline shift potential due to unilateral canine loss
4. Crowded arches accompanied with extreme proclination
5. Abnormal primary canine root resorption
6. Lack of developmental spacing
7. Anomalies such as ankylosis, ectopic eruption.
Contraindications
1. Mild to moderate crowding (about 8 mm or less).
2. Congenital absence of teeth providing space.
3. Where extensive caries of permanent first molars requires their removal,
4. Accompanying deep or open bites without correction.
5. Severe class II, III of dental/ skeletal origin.
6. Cleft lip and palate cases.

Ans.181
Composition
• Stainless steel crowns (18/8)- Austentic type of alloy is used e.g. Rocky mountain and Unitek.
17-19% chromium
10-13% nickel
67% iron and
4% minor elements.
The austentic types provide the best corrosion resistance of all the stainless steel.
These crowns are available in various sizes to suit the morphology of the tooth being restored. Mostly these crowns are used in the
posterior teeth which have undergone pulp therapy. The commonly used sizes are 3, 4 and 5
• Nickel-base crowns are Inconel 600 type of alloy.
72% nickel
14% chromium
6-10% Fe
0.04% carbon
0.35% manganese and
0.2% silicon.

Ans.182
• Plane I: Moderate Sedation and Analgesia
Achieved with concentration of 5-25% N2O (95-75%)
• Plane II: Dissociation Sedation and Analgesia
Concentration of 25-45%
• Pane III: Total Anesthesia (Analgesia)
Achieved with 45-65% concentrations
• Pane IV:
Many develop beyond 65-85%
Light anesthesia – contact with patient lost.

Ans.183
Ropivacaine is a long-acting amide anesthetic, similar to bupivacaine and etidocaine induration of activity. It is similar structurally to
mepivacaine and bupivacaine but is unique in that ropivacaine is prepared as an isomer rather than a racemic mixture. Ropivacaine
has a greater margin of safety between convulsive and lethal doses than does bupivacaine and also a lower dysrhythmogenic potential
than bupivacaine. Ropivacaine has demonstrated decreased cardiotoxicity relative to bupivacaine.

Ans.184
Cleft lip: Various theories have been suggested to explain the development of the usual cleft lip:
• Failure of fusion between median nasal process and maxillary process (Dursy-His hypothesis).

www.targeteducare.com info@targeteducare.com
Helpline: 93 2233 6677 (Working days 11 AM to 6 PM)
TARGET EDUCARE PVT. LTD.
AIPGEE - 1 Explanations
• Failure of mesodermal migration between the two layered epithelial membrane which results due to fusion between the two
processes. This eventually leads to a breakdown and cleft formation (Fleischmann, Veau and Stark).
• Rupture of cyst formed at the site of fusion.

Ans.185
Classification based on neuromuscular involvement-
1. Spasticity
- Approximately seen in 70% of cases
- Involvement of cerebral cortex
2. Athetosis
- Approximately in 15% of cases
- Involvement of basal ganglia
3. Ataxia
- Involved muscles unable to contract completely
- Involvement of cerebellum
4. Mixed

Ans.186
Oral changes
• Hyperplasia of the marrow, leading to expansion of the marrow cavity.
• Chipmunk facies with an enlargement of the maxilla and spacing of the teeth, increased overjet, protrusion of the middle-
third of the face, extreme tightness of the upper lip.
• Ocular hypertelorism, epicanthal folds and browning of the skin.
• Radiographic changes: ‘chicken wire’ appearance of the alveolar bone, delayed pneumatisation of the sinuses, hair-on-end
appearance of the skull.
• Large medullary cavities, thin cortices and generalized osteoporosis.
• Pain and swelling of the parotid glands and atrophic candidiasis.

Ans.187
HIV is sensitive to heat for 20 minutes but autoclaving at 1210C for 15 minutes at 1 atmospheric pressure or dry heating of
instruments up to 1700C kills this virus. At room temperature it can survive for about 7 days. The virus can be inactivated by heating
lyophilized factor at 680C for 72 hours, liquid plasma at 600C for 10 hrs.
Disinfectants for innate objects: Chemical disinfectants must not be used for needles and syringes. Before immersion of innate objects
on chemical disinfectant remove gross contamination on the instruments like dried blood because disinfectant will not penetrate into
these. The common disinfectants that are used are:
• 0.2% sodium hypochlorite
• Calcium hypochlorite
• 6% hydrogen peroxide for more than 30 minutes
• 2% glutaraldehyde and 6% hydrogen peroxide combination
• Sodium dichloroisocyanate.
Disinfectants for living tissues are:
• Povidine
• 2% propranol
• Ethanol
• HIV is completely inactivated by treatment for 10 minutes at room temperature with 10% household bleach, 50% ethanol,
35% isopropanol, 0.5% paraformaldehyde or 3% hydrogen peroxide.
Gloves may be autoclaved or disinfected by immersing them in boiling water for 20 minutes. Alternatively, overnight soaking of
1% sodium hypochlorite may be done.

Ans.188
Orofacial features that help in diagnosis of syndromes
Special Features Syndromes
Mental retardation Down S, William’s S, Angelman S, Fragile-X S, Prader Willi S, Rubinstein –Taybi S
Cerebral palsy Lesch-Nyhan S, Shanken Baby S, Sturge-Weber S
Epilepsy Ohtahara S, West S, Dravet’s S, Lennox-Gastaut S, Landau-Kleffner S
Learning disability Kanner’s S (autism), Asperger’s S
Deafness Usher S, Pendred S, Waaderburg S
Blindness Usher S, Stevens-Johnson S

www.targeteducare.com info@targeteducare.com
Helpline: 93 2233 6677 (Working days 11 AM to 6 PM)
TARGET EDUCARE PVT. LTD.
AIPGEE - 1 Explanations
Ans.189
• Endoflas contain:
- Zinc oxide 56.5%
- Barium sulfate 1.63%
- Iodoform 40.6%
- Calcium hydroxide 1.07%
- Eugenol
- Pentachlorophenol

Ans.190
Measurement data can be presented graphically in a number of different ways, and frequently, it is possible to clarify growth changes
by carrying the method of display. Growth data can be presented by plotting the size attained s function of age, which is called a
“distance” curve, or as a “velocity” curve, showing not the total length but the increment added each year. Changes in the rate of
growth are much more easily seen in the velocity curve than the distance curve.

Ans.191
Dental ages 13, 14 and 15 are characterized by the extent of completion of the roots permanent teeth. By dental age 15, if a third
molar is going to form, it should be apparent on the radiographs, and the roots of all other permanent teeth should be complete.

Ans.192
Simple clinical tests for mouth breathing can also be misleading. The highly vascular nasal mucosa undergoes cycles of engorgement
with blood and shrinkage. The cycles alternate between the two nostrils: when one is clear, the other is usually somewhat obstructed.
For this reason, clinical tests to determine whether the patient can breathe freely through both nostrils nearly always show that one is
at least partially blocked. One partially obstructed nostril should not be interpreted as a problem with normal nasal breathing.

Ans.193
Occasionally the molar occlusion is Class II on one side, and Class I on the other. Angle called this a Class II subdivision right or left,
depending on which was the Class II side. In modern classification the subdivision label rarely is useful because it does not describe
the real problem. The asymmetric molar relationship reflects either an asymmetry within one or both the dental arches (typically due
to loss of space when one primary second molar was lost prematurely), or a haw discrepancy of the jaw or dentition. These must be
distinguished and should already have been addressed in the first or second steps in the classification procedure.

Ans.194
In the early 1900s the German physiologist Wolff demonstrated that bone trabeculae were arranged in response to the stress lines on
the bone. The internal architecture of the head of the femur is the classic example, but the condylar process of the mandible shows the
same effect of “Wolff’s law of bone”. This led Angle to two key concepts. The first was that skeletal growth could be influenced
readily by external pressures. If bone remodeled when stressed, the etiology of Class II or Class III problems must be abnormal
stresses on the jaws, but different patterns of pressure associated with treatment could change growth so as to overcome the problem.
Angle came to believe that skeletal structures were so adaptable that just rubber bands connecting the upper to the lower teeth could
overcome improper jaw relationships, stimulating growth where it was needed.

Ans.195
In infants, the segments can be repositioned surprisingly quickly and easily, so that the period of active treatment is a few weeks at
most. If pre-surgical movement of maxillary segments is indicated, this typically would be done beginning at 3 to 6 weeks of age, so
that the lip closure could be carried out at approximately 10 weeks. A passive palate, similar to an orthodontic retainer, is then used
for a few months after lip closure.

Ans.196
• CAT performs well:
 Mild-moderate crowding with IPR or expansion
 Posterior dental expansion
 Close mild-moderate spacing
 Absolute intrusion (r or z teeth only)
 Lower incisor extraction for severe crowding
 Tip molar distally
• CAT does not perform well:
 Dental expansion or blocked-out teeth
 Extrusion of incisors*
www.targeteducare.com info@targeteducare.com
Helpline: 93 2233 6677 (Working days 11 AM to 6 PM)
TARGET EDUCARE PVT. LTD.
AIPGEE - 1 Explanations
 High canines
 Severe rotations (particularly of round teeth)
 Leveling by relative intrusion
 Molar uprighting (any teeth with large undercuts)
 Translation of molars*
 Closure of premolar extraction spaces

Ans.197
Dental anterior crossbites typically develop as the permanent incisors erupt. Those diagnosed after overbite is established require
appliance therapy for correction. The first concern is adequate space for tooth movement, which usually requires bilateral disking,
extraction of the adjacent primary teeth, or opening space for tooth movement. The diagnostic evaluation should determine whether
tipping will provide appropriate correction. Often it will, because the problem arose as eruption paths were deflected. If teeth are
tipped when bodily movement is required, stability of the result is questionable In a young child, the best method for tipping maxillary
and mandibular anterior teeth out of crossbite is a removable appliance using finger springs for facial movement of maxillary incisors
or (less frequently) an active labial bow for lingual movement of mandibular incisors. Two maxillary anterior teeth can be moved
facially with one 22 mil double helical cantilever spring. The appliance should have multiple clasps for retention, but a labial bow is
usually contraindicated because it can interfere with facial movement of the incisors and would add little or no retention.

Ans.198
• Class III functional appliance for excessive mandibular growth make no pretense of restraining mandibulat growth. They are
designed to rotate the mandible down and back and to guide the eruption of the teeth so that the upper posterior teeth erupt down
and forward while eruption of lower teeth is restrained. This rotates the occlusal plane in the direction that favors correction of a
class III molar relationship. These appliance also tip the mandibular incisors lingually and the maxillary incisors facially,
introducing an element of dental camouflage for the skeletal discrepancy. The only difference from a functional appliance for a
maxillary deficiency patient is the absence of lip pads.
• Although the theory of the functional appliance is quite different from that of the chin cup, the treatment effects are very similar,
and the two approaches are approximately equally effective (or, in severe cases, equally ineffective).

Ans.199
In summary, retention is needed for all patients who had fixed orthodontic appliances to correct intra-arch irregularities. It should be:
Essentially full-time for the first 3 to 4 months, except that the retainers not only can but should be removed while eating (unless
periodontal bone loss or other special circumstances require permanent splinting). Continued on a part-time basis for at least l2
months, to allow time for remodeling of gingival tissues, if significant growth remains, continued part-time until completion of
growth. For practical purposes this means that nearly all patients treated in the early permanent dentition will require retention of
incisor alignment at least until their late teens, and in those with skeletal disproportions initially, part-time use of a functional
appliance or extraoral force probably will be needed.

Ans.200
For other adult patients, with one important exception: if the maxilla was expanded transversely it is critically important not only to
maintain the expansion during the finishing orthodontics, but also to have full-time retainer wear in the maxilla for at least 6 months.
If a transpalatal lingual arch was placed following surgery, it should not be removed during the first postsurgical year.

www.targeteducare.com info@targeteducare.com
Helpline: 93 2233 6677 (Working days 11 AM to 6 PM)

You might also like